What is the common difference and the 5 th term in the arithmetic formula for an=6n+3

Answers

Answer 1

Answer:

is an=6n+3 your question?

Step-by-step explanation:

Answer 2

Answer:

Common Difference (d) =6

5th term = 33

Step-by-step explanation:

[tex]a_n= 6n +3.... (given) \\

Plug\: n = (n +1)\\

a_{(n+1)}= 6(n+1) +3 = 6n + 6 +3 = 6n + 9\\

Common \: Difference \: (d) = a_{(n+1)} - a_n\\

= 6n + 9 - (6n+ 3)\\

= 6n + 9 - 6n - 3\\

= 9-3\\

\red{\boxed{\bold {Common \: Difference \: (d) =6}}} \\

Plug \: n = 5 \: in\: a_n= 6n +3\: we \: find:\\

a_5= 6\times 5+3= 30 + 3 = 33\\

\huge\purple {\boxed {a_5= 33}} \\[/tex]


Related Questions

I need help with this please

Answers

Answer:

Step-by-step explanation:

For (a), you will use that 2 points that are closest to lying on the line which are the points located at (1, 14) and (7, 7).

For (b), you will use those 2 points to find the slope of the line using the slope formula:

[tex]m=\frac{7-14}{7-1}=\frac{-7}{6}=-1.167[/tex]

For (c), you will use point-slope form to write the equation. Point-slope form is

[tex]y-y_1=m(x-x_1)[/tex] where x and y stay x and y in the equation and x1 and y1 are replaced with one of the coordinates. Let's use (7, 7). Keep in mind that IT DOESN'T MATTER WHICH POINT YOU PICK...YOU WILL GET THE SAME EQUATION WITH EITHER ONE! And this is because both those points lie on the same line...the line for which we will write the equation.

We have m = -1.167, y = 7 and x = 7:

y - 7 = -1.167(x - 7)

That's the point-slope form of the line, but rarely is it ever left in that form. I've only seen it left in point-slope form in calculus. Most of the time, from point-slope form, you are asked to put it into slope-intercept form, and here is no exception. Putting the equation into slope-intercept form is the same thing as solving it for y. So let's get y all by itself on one side of the equals sign and everything else over on the other side. We also of course need to distribute into the parenthesis:

y - 7 = -1.167x + 8.169 and

y = -1.167 + 8.169 + 7 so

y = -1.167 + 15.169

That's your equation in slope-intercept form, so you're done!

Marsha was shopping for a new dishwasher and researched the different models she could afford. Using Marsha’s frequency distribution table, determine the median price for dishwashers. Price,p($) Frequency,f 250 2 275 4 280 1 290 2 310 6 315 2 320 1 325 7 330 1 335 1 340 1 350 2 ​

Answers

Answer

310

Step-by-step explanation:

add them all up and divide by 12

Two similar triangles are shown below: Two triangles are shown. The sides of the triangle on the left are marked 6, 4, 3. The sides of the triangle on the right are marked as 3, 2 and 1.5. For the triangle on the left, the angle between sides marked 4 and 6 is labeled as w, marked by a double arc, and the angle between the sides marked 6 and 3 is labeled as x, marked by a single arc. The third angle is marked by a triple arc. For the triangle on the right, the angle between sides marked 3 and 1.5 is labeled as y and the angle between the sides marked 2 and 3 is labeled as v, marked by a double arc. The angle between the sides 2 and 1.5 is labeled as z, marked by a triple arc, and it is also the angle on the top vertex of this triangle. Which two sets of angles are corresponding angles? (4 points) a ∠w and ∠v; ∠x and ∠y b ∠w and ∠y; ∠x and ∠v c ∠w and ∠z; ∠x and ∠v d ∠w and ∠z; ∠x and ∠y

Answers

Answer:

a.  ∠w and ∠v; ∠x and ∠y

Step-by-step explanation:

We can see that the scale factor is 2

Corresponding sides have length of:

6 and 3;  4 and 2;  3 and 1.5

Therefore corresponding angles, opposite to corresponding sides are:

∠x and ∠y; ∠w and ∠v

Correct answer option:

a.  ∠w and ∠v; ∠x and ∠y

Answer:

His answer is correct!

Step-by-step explanation:

Got it right

Find the midpoint of the segment with the following endpoints.
(-1,9) and (-7,1)

Answers

Answer:

1 1/3

Step-by-step explanation:

m=y2-y1/x2-x1

or rise over run

m=1-9/(-7)-(-1)= (-8)/(-6)= 1 1/3

Answer:

11/3

Step-by-step explanation:

A company’s sales increased 17% this year, to $8267. What were their sales last year?

Answers

Answer:

eekk im not shure sorry but it might be 8156 i no no

Step-by-step explanation:

curtis divided 27.16 by 2.8 in Got 0.97 is his answer reasonable why or why not

Answers

Given :

Curtis divided 27.16 by 2.8 and got 0.97 .

To Find :

Is his answer reasonable why or why not.

Solution :

When we divide 27.16 by 0.97 we will get 9.7.

So, Curtis did a calculation error of decimal and his answer is not acceptable.

Hence, this is the required solution.

study of class attendance and grades among first-year students at a state university showed that, in general, students who missed a higher percent of their classes earned lower grades. Class attendance explained 36% of the variation in grade index among the students. What is the numerical value of the correlation between percent of classes attended and grade index?

Answers

Yess yes yes yes ffnjtssnm

The digit 5 in which number represents a value of 0.5? The answers are 5.4, 0.059, and 8.513.

Answers

Answer:

8.513 is the answer

Step-by-step explanation:

The value of 5 in 0.5 is the tenths place, hence if we look at all other options, the values of 5 are 5(ones place), and 0.05(Hundredth), so the option of 8.513 suits the question's answer. Hope this helped, Thank you.

There were 40 students who auditioned for the school play, 3/4 of them were called back for a second audition. How many students were called back for another audition?

Answers

Answer:

30 students

Step-by-step explanation:

Number of students called back for second audition

[tex] = \frac{3}{4} \times 40 \\ \\ = 3 \times 10 \\ = 30[/tex]

Answer:

30 students were called back for audition

Step-by-step explanation:

every 1/4=10 student so 3/4 will be 30 students altogether

3. Fill in the blanks so that the answer to this number sentence is correct: 2ab​

Answers

Answer:

where's the question or the picture of the question

If the length of a box of Milk Duds box is tripled, how much longer will the box be?​

Answers

Answer:

3x

Step-by-step explanation:

x = original length of the box.

~theLocoCoco

The graph of f(x)=2^x is shown in black (k). Match each transformation of this function with a graph below.

Answers

Answer:

Step-by-step explanation:

From the graph attached,

Black curve represents a function f(x) = 2ˣ

When this curve is reflected over the x-axis image of the function will be,

g(x) = -[tex]2^{x}[/tex] [Red curve]

Similarly, curve represented by h(x) = [tex]2^{-x}[/tex] is the blue curve [image of f(x) reflected over y-axis]

When this curve is reflected over the x-axis, image of the function will be,

h'(x) = [tex]-2^{-x}[/tex] [green curve]

[tex]-2^{x}[/tex] → Red

[tex]-2^{-x}[/tex] → Green

[tex]2^{-x}[/tex] → Blue

What are the coordinates of the image of (- 4, 5) under the translation (x,y) (x-3,y+5)

Answers

(-4-3) (5+5) = -7,10

______is 15% of $84,652?

Answers

Answer: $12,697.80

Step-by-step explanation:

To find 15% of $84,652, you want to multiply them together.

0.15×84,652=12,697.80

Now, we know that $12,697.80 is 15% of $84,652.

Rewrite the following improper fraction as a decimal 15/2

Answers

Answer:

7.5

Step-by-step explanation:

Answer:

7.5

Step-by-step explanation:

all you need to do is divide 15 by 2

this is a segment addition practice. can someone please help me find the length indicated?

Answers

Answer:

3

Step-by-step explanation:

Points C, D, and E are collinear.

Segment CD = 8

Segment CE = 11

Segment DE = ?

According to the segment addition postulate:

[tex] CD + DE = CE [/tex]

[tex] 8 + DE = 11 [/tex] (substitution)

Subtract 8 from both sides

[tex] 8 + DE - 8 = 11 - 8 [/tex]

[tex] DE = 3 [/tex]

The missing lenght indicated = 3

Jamila paid $6 for a movie ticket, $1.59 for a small box of popcorn, and $2.50 for a drink at a movie theater. Which operation(s) would help Jamila find the total amount she spent at the theater?

Answers

Answer:

addition

Step-by-step explanation:

Answer:

It would be addition since you can see thats litterly the problem

Step-by-step explanation:

Find the remaining side of a 45° – 45° – 90° triangle if the shorter sides are 2/3 each .

Answers

Answer:

The length of the longer side is 2√2/3

Step-by-step explanation:

A 45-45-90 triangle is a right-angled triangle.

In fact, since it has two angles equal, it is a right angled isosceles triangle. What we are saying is that two of its sides are equal.

Hence, we have two short sides which are the opposite and the adjacent and a longer side which is the hypotenuse

Thus, to find the length of the longer side, we employ Pythagoras’ theorem and it states that the square of the hypotenuse equals the sum of the squares of the two other sides;

Mathematically, let’s call the longer side x

so x^2 = (2/3)^2 + (2/3)^2

x^2 = 4/9 + 4/9

x^2 = 8/9

x = √(8/9)

x = 2√2/3

The sum of two complex numbers, where the real numbers do not equal zero, results in a sum of 34i. Which statement must be true about the complex numbers?

Answers

Answer:

The complex numbers have opposite real numbers.

Answer:

The Complex Numbers Have Opposite Real Parts

Step-by-step explanation:

The perimeter of the figure is 18x +2. One side is 3x − 3. Find the length of the indicated side.

Answers

Answer:

Step-by-step explanation:

3000000000

X2 - X - 12
what's the answer for this question i am stuck

Answers

Answer:

not enough info

Step-by-step explanation:

not enough info

Jordan’s credit card has an APR of 10.59%, compounded monthly. He is required to make a minimum payment of 3.96% of his current balance every month. At the beginning of March, Jordan had a balance of $628.16 on his credit card. The following table shows his credit card purchases over the next few months. Month Cost ($) March 50.81 March 48.04 April 77.36 April 32.40 April 49.20 May 25.79 May 79.39 May 79.08 If Jordan makes only the minimum monthly payments in March, April, and May, what will his balance be after he makes the minimum payment for May? (Assume that interest is compounded before the monthly payment is made, and that the monthly payment is applied at the end of the month. Round all dollar values to the nearest cent.) a. $1,094.10 b. $988.97 c. $967.60 d. $1,070.23

Answers

Answer:

b. 988.97

Step-by-step explanation:

got a 100 on the test

The balance at the end of May after the minimum payment is made is

$571.64.

What is compound interest?

It is the interest we earned on the interest.

The formula for the amount earned with compound interest after n years is given as:

A = P [tex](1 + r/n)^{nt}[/tex]

P = principal

R = rate

t = time in years

n = number of times compounded in a year.

We have,

P = $628.16

r = 10.59%

n = 12

t = 1/12

The amount after March.

A = 628.16 [tex](1 + 0.1059)^{12\times0.08[/tex]

A = $633.48

Monthly payment.

= 3.96/100 x 633.48

= $25.09

Remaining amount.

= 633.48 - 25.09

= $609.39

The amount after April.

A = 609.39 [tex](1 + 0.1059)^{12\times0.08[/tex]

A = $614.55

Monthly payment.

= 3.96/100 x 614.55

= $24.34

Remaining amount.

= 614.55 - 24.34

= $590.21

The amount after May.

A = 590.21 [tex](1 + 0.1059)^{12\times0.08[/tex]

A = $595.21

Monthly payment.

= 3.96/100 x 595.21

= $23.57

Remaining amount.

= 595.21 - 23.57

= $571.64

Thus,

The balance at the end of May is $571.64.

Learn more about compound interest here:

https://brainly.com/question/13155407

#SPJ3

Evaluate the expression 43+ [(96 = 8) ~ 5] - 42.

Answers

Answer:

233

Step-by-step explanation:

Step-by-step explanation:

43+[(96/8=12)×5]-42

=43+60-42

=103-42

=61

Translate the following word phrase into an algebraic expression: nine times the difference of 5 and y.

Answers

Hey there! I'm happy to help!

Difference means that you subtract.

5-y

And we multiply this entire thing by 9, giving us 9(5-y).

Have a wonderful day! :D

please help!!! urgent

Answers

Answer:

E

Step-by-step explanation:

is the answer

Whats the answer for −2 1/2,−5 3/4 The distance between the two numbers is

Answers

Answer:

3 1/4

Step-by-step explanation:

-5 3/4 + 3 1/4 = -2 1/2

Markel said he has 4 times
as many yellow cars as
blue cars. What is the ratio
for blue cars to yellow
cars?

Answers

Answer:

4:1

Step-by-step explanation:

33 feet is how many meters? Hint: 1 ft ≈ 0.305 m Round your answer to the nearest tenth. (HELP!)

Answers

Answer:

10.1m

Step-by-step explanation:

1ft = 0.305m

33ft = 33 x 0.305m = 10.065m ≈ 10.1m

33 ft because it’s 33 ft

20. Two employees of a company have to file expense reports for their travel. The first had taken 7 trips with a mean trip cost of $750. The second took a total of 5 trips with a mean cost of $1200. The accounting department wants to determine the mean cost of all the trips that these two employees took. What is the mean cost of all 12 trips? Proposed Solution: Since we have two costs (one for each employee) we can find the mean cost of both employees by the following computation: (750 + 1200) / 2 = 975 What was done wrong in the proposed solution?

Answers

Since there are 7 trips with a mean cost 750 and 5 trips with mean cost 1200, the calculation should be [tex](7\times 750+5\times 1200) \div 12[/tex]

The calculation is as follows:

We know that

Mean cost =  Total cost ÷  total number of trips

Here, Total number of trips(observation ) =  7+5 = 12

Total cost = total cost of two employees

total cost of first employee = [tex]7\times 750[/tex]

And, total cost of second employee = [tex]5\times 1200[/tex]

So,

Mean cost of 12 trips = [tex](7\times 750+5\times 1200) \div 12[/tex] = 937.50

Learn more: https://brainly.com/question/6269728?referrer=searchResults

which operation must you use to calculate the submarine's change in elevation? which word or words in the problem signify this operation?

Answers

The order of operations

Answer:

The problem states that the submarine dives down 115 feet deeper. The words dives and down convey that the submarine dove farther below its initial level. So, I should subtract 115 feet from -502.5 feet.

Step-by-step explanation:

Correct answer

Other Questions
which countrys were communists during the vietnam war yesterday jack drove 33 1/2 miles. He used 1 1/4 gallons of gasloine Commercial hotels are usually in downtown or _____________ districts. Find the probability for one roll of a number cube.P(number is not 5)A. 0B. 5/6C. 1/6D. 1 Sally used 3/4 cup of milk to make pancakes, 5/8 cup of milk to make scrambled eggs, and 1/2 cup of milk to make biscuits. How many cups of milk did Sally use altogether? What is the formula to determine Target Heart Rate Zone? Is 0.843 less than 0.846 3/4 + (-2 1/2) on a number line.. select the location on the number line to plot the sum.. HURRYYYYY Which common item is made from copper?soapcoinsbaby powdercolored pencils Maria es no alta; es muy what is the value of y? i don't get it (solve) What Is The Distance Between - 20 And - 6 On A Number Line ? . an 8 degree rise in temperature I need with the basic understanding of this 1. Which structure is not part of the endomembrane system?(A) nuclear envelope(C) Golgi apparatus(B) chloroplast(D) plasma membrane A good witness is someone who ______ give an answer too quickly.Brain Games Season 1 Eps 3 Match the story element with its definition. Column A 1. Setting: Setting 2. Plot: Plot 3. Conflict: Conflict 4. Antagonist: Antagonist 5. Protagonist: Protagonist 6. Exposition: Exposition 7. Inciting Incident: Inciting Incident 8. Rising Action: Rising Action 9. Climax: Climax 10. Falling Action: Falling Action 11. Resolution: Resolution Column B a.The main part of the story, leading to the climax b.The chain of events that happen in a story c.The most exciting part of the story d.Te end of the story, loose ends are tied up e.The character or force in conflict with the protagonist f.The first part of the plot, characters and setting are revealed g.The main character h.Problems that arise in a story i.Events that follow the climax and lead to the resolution j.The event that sets the story in motion k.The time and place of a story A letter in the word ACCEL is chosen. What is the theoretical probability of choosing a C? The acceleration due to gravity on the Moon is gM. Suppose an astronaut on the Moon drops an object from a height of H. The time it would take the object to reach the Moons surface would be TM. The same object is dropped from the same height on Earth, where the acceleration due to gravity is gE. The time it takes the object to reach the Earths surface is TE. Which of the following is a correct mathematical relationship for the two times?TE=gEgMTMTE=gEgMTMATE=gMgETM TE=gMgETMBTE=gEgMTMTE=gEgMTMCTE=gMgETMTE=gMgETMDTE=TM